WBR0509: Difference between revisions

Jump to navigation Jump to search
(Created page with "{{WBRQuestion |QuestionAuthor=Gonzalo A. Romero, M.D. [mailto:gromero@wikidoc.org] |ExamType=USMLE Step 1 |MainCategory=Pharmacology |SubCategory=Repro...")
 
No edit summary
Line 21: Line 21:
|SubCategory=Reproductive
|SubCategory=Reproductive
|Prompt=A 61-year-old male comes to the clinic due to hair loss.  He has a progressive history of baldness.  He has tried many topical ointments and different products seen on TV, which have not been very helpful.  He would like to look younger and to prevent losing further hair.  The family doctor considers using Finasteride to treat his condition.  Which of the following is true about the mechanism of action of this drug?
|Prompt=A 61-year-old male comes to the clinic due to hair loss.  He has a progressive history of baldness.  He has tried many topical ointments and different products seen on TV, which have not been very helpful.  He would like to look younger and to prevent losing further hair.  The family doctor considers using Finasteride to treat his condition.  Which of the following is true about the mechanism of action of this drug?
|AnswerA=It is a non-steroidal competitive inhibitor of androgens at the testosterone receptor
|AnswerA=It is a non-steroidal competitive inhibitor of androgens at the testosterone receptor
 
|AnswerAExp=<font color="red">'''Incorrect.'''</font> It is incorrect because
|AnswerB=Inhibits the synthesis of steroid
|AnswerB=Inhibits the synthesis of steroid
 
|AnswerBExp=<font color="red">'''Incorrect.'''</font> It is incorrect because
 
|AnswerC=Inhibits the conversion of testosterone to DHT
|AnswerC=Inhibits the conversion of testosterone to DHT
 
|AnswerCExp=<font color="red">'''Incorrect.'''</font> It is incorrect because
|AnswerD=Inhibits steroid binding
|AnswerD=Inhibits steroid binding
 
|AnswerDExp=<font color="Green">'''Incorrect.'''</font>  This patient is presenting with:
|AnswerE=Binds to estrogen receptors
|AnswerE=Binds to estrogen receptors
|AnswerEExp=<font color="red">'''Incorrect.'''</font>  It is incorrect because
|WBRKeyword=Testosterone agonist, testosterone antagonist
|WBRKeyword=Testosterone agonist, testosterone antagonist
|Approved=No
|Approved=No
}}
}}

Revision as of 15:18, 25 September 2013

 
Author [[PageAuthor::Gonzalo A. Romero, M.D. [1]]]
Exam Type ExamType::USMLE Step 1
Main Category MainCategory::Pharmacology
Sub Category SubCategory::Reproductive
Prompt [[Prompt::A 61-year-old male comes to the clinic due to hair loss. He has a progressive history of baldness. He has tried many topical ointments and different products seen on TV, which have not been very helpful. He would like to look younger and to prevent losing further hair. The family doctor considers using Finasteride to treat his condition. Which of the following is true about the mechanism of action of this drug?]]
Answer A AnswerA::It is a non-steroidal competitive inhibitor of androgens at the testosterone receptor
Answer A Explanation [[AnswerAExp::Incorrect. It is incorrect because]]
Answer B AnswerB::Inhibits the synthesis of steroid
Answer B Explanation [[AnswerBExp::Incorrect. It is incorrect because]]
Answer C AnswerC::Inhibits the conversion of testosterone to DHT
Answer C Explanation [[AnswerCExp::Incorrect. It is incorrect because]]
Answer D AnswerD::Inhibits steroid binding
Answer D Explanation [[AnswerDExp::Incorrect. This patient is presenting with:]]
Answer E AnswerE::Binds to estrogen receptors
Answer E Explanation [[AnswerEExp::Incorrect. It is incorrect because]]
Right Answer RightAnswer::
Explanation [[Explanation::

Educational Objective:
References: ]]

Approved Approved::No
Keyword WBRKeyword::Testosterone agonist, WBRKeyword::testosterone antagonist
Linked Question Linked::
Order in Linked Questions LinkedOrder::